Search found 876 matches


Meaning that if I were an instructor, I would rate this essay as 5.5 because your recommendations could have been more stronger. The current recommendations are fine but I would suggest you focus for 6.6 because you have that potential.

by ketkoag

Tue Apr 19, 2011 7:10 pm
Forum: GMAT Essays (AWA)
Topic: Please rate my 1st essay...tried for 2 months...Analyze argu
Replies: 3
Views: 1294

The argument of the passage states that the installation of computerized warning systems in planes will help avoid midair collisions. This system depends on signals that may be transmitted ahead of time before planes are en route a course to crash into other planes. The argument assumes that the war...

by ketkoag

Sun Apr 17, 2011 6:53 pm
Forum: GMAT Essays (AWA)
Topic: Analyze the Argument (OG 12th ed.) Please read/rate! Thanks
Replies: 1
Views: 1658

Nice essay! You've mentioned Secondly twice in the second paragraph. Could've structured it better. But no big deal! In last paragraph, instead of "neither specifies", use "specifies neither". And read this sentence again. You'll find the mistake. But overall good essay. Definite...

by ketkoag

Sun Apr 17, 2011 6:36 pm
Forum: GMAT Essays (AWA)
Topic: Please rate my 1st essay...tried for 2 months...Analyze argu
Replies: 3
Views: 1294

para 3 never states that "Managers’ intuition works contrary to their rational and analytical skills." what it implies is that intuition sometimes helps the managers in taking their decisions. and that is why para 3 explains the different ways in which the manager use intuition. E i...

by ketkoag

Thu Nov 26, 2009 4:19 am
Forum: Reading Comprehension
Topic: 1000 RC passage 13 #7
Replies: 2
Views: 2005
by ketkoag

Thu Sep 10, 2009 10:21 am
Forum: Critical Reasoning
Topic: LSAT Problems
Replies: 4
Views: 2772

Re: LSAT

The workers at Bell Manufacturing will shortly go on strike unless the management increases their wages. As Bell’s president is well aware, however, in order to increase the worker’s wages, Bell would have to sell off some of its subsidiaries. Since, The workers at Bell Manufacturing will not b...

by ketkoag

Thu Sep 10, 2009 10:09 am
Forum: Critical Reasoning
Topic: LSAT
Replies: 12
Views: 4309

C it is.. C is definitely the conclusion coz it directly fills the unstated premise that is necessary for this argument to be valid..

by ketkoag

Thu Sep 10, 2009 9:00 am
Forum: Critical Reasoning
Topic: LSAT
Replies: 12
Views: 4309

gmatprep

how B is sufficient?

by ketkoag

Sun Sep 06, 2009 9:19 am
Forum: Data Sufficiency
Topic: gmatprep
Replies: 1
Views: 1367

gmatprep

please explain this one as well.

by ketkoag

Sun Sep 06, 2009 9:19 am
Forum: Data Sufficiency
Topic: gmatprep
Replies: 3
Views: 1535

gmatprep

i assumed that both the triangles are isoceles triangles and then used area of the triangles.
please explain whether i assumed correctly..

by ketkoag

Sun Sep 06, 2009 9:15 am
Forum: Data Sufficiency
Topic: gmatprep
Replies: 2
Views: 1299

gmatprep

please explain how to solve this one?
my method..
take 1. since 1/(k-1) > 0, k-1 should be positive.
therefore, k-1 < 1
hence, k < 2 insuff.

take 2. since 1/(k+2) > 0 , k+1 should be positive.
therefore, k+1 < 1
hence k<0 suff.

hence B..
please lemme know what i did wrong??

by ketkoag

Sun Sep 06, 2009 9:07 am
Forum: Data Sufficiency
Topic: gmatprep
Replies: 2
Views: 1405

nice explanation, thanks

by ketkoag

Sun Sep 06, 2009 6:39 am
Forum: Data Sufficiency
Topic: prep
Replies: 2
Views: 1368

prep

please explain this one

by ketkoag

Sat Sep 05, 2009 12:14 pm
Forum: Data Sufficiency
Topic: prep
Replies: 2
Views: 1368

D is the answer coz it is out of scope.. it doesn't matter whether the actress has given nice reviews about it.. it is totally out of scope.. we can eliminate all the other answer choices because they are in the scope of the question to some extent.. In critical reasoning it doesn't matter whether t...

by ketkoag

Sat Sep 05, 2009 8:33 am
Forum: Critical Reasoning
Topic: major film studio
Replies: 10
Views: 3344

i think A weakens the argument coz the argument says that the course has an effect on the inmates and thus inmates who take the course are less likely to commit crime..
A says that course doesn't have any effect on the inmates..

by ketkoag

Sat Sep 05, 2009 8:20 am
Forum: Critical Reasoning
Topic: discuss hardest assumption question.
Replies: 11
Views: 2196